Antiderivative of a rational function

Click For Summary
The discussion focuses on finding the antiderivative of a rational function using partial fraction decomposition. The original integral involves a numerator of degree 3 and a denominator of degree 4, prompting the use of partial fractions. A correction is noted that the last term in the decomposition should include a first-order polynomial in the numerator, which is necessary due to the degree of the denominator. The coefficients are determined to be A = 0, B = 1, C = 2, and D = -1, resolving earlier contradictions in calculations. Understanding the need for an additional term in the numerator is essential for accurately matching polynomial degrees and coefficients.
nuuskur
Science Advisor
Messages
926
Reaction score
1,226

Homework Statement


Consider the integral:
\int\frac{2x^3 -4x^2 +8x +7}{(x-1)^2 (x^2 +4x +8)}{\rm{d}}x

Homework Equations

The Attempt at a Solution


The degree of the denominator is 4 and the numerator's is 3, hence I thought I would try partial fractions:
\frac{A}{x-1} +\frac{B}{(x-1)^2} +\frac{C}{x^2 +4x +8} = \frac{2x^3 -4x^2 +8x +7}{(x-1)^2 (x^2 +4x +8)}multiplying both sides by the denominator on the right side we would have:
A(x-1)(x^2 +4x +8) +B(x^2 +4x +8) +C(x-1)^2 = 2x^3 -4x^2 +8x +7\\Ax^3 +(3A +B +C)x^2 +(4A +4B -2C)x -(8A -8B -C) = 2x^3 -4x^2 +8x +7
So I should be able to conclude that A = 2, however, the problem is that on one hand I get that 3B = -10 and on the other hand, 10B = 23. Have I made a mistake in the calculations? Is any such rational function divisible [not sure if that's the correct word] into partial fractions?
Is there any other method for tackling such a problem?

Thank you in advance.
 
Last edited:
Physics news on Phys.org
nuuskur said:

Homework Statement


Consider the integral:
\int\frac{2x^3 -4x^2 +8x +7}{(x-1)^2 (x^2 +4x +8)}{\rm{d}}x

Homework Equations

The Attempt at a Solution


The degree of the denominator is 4 and the numerator's is 3, hence I thought I would try partial fractions:
\frac{A}{x-1} +\frac{B}{(x-1)^2} +\frac{C}{x^2 +4x +8} = \frac{2x^3 -4x^2 +8x +7}{(x-1)^2 (x^2 +4x +8)}multiplying both sides by the denominator on the right side we would have:
A(x-1)(x^2 +4x +8) +B(x^2 +4x +8) +C(x-1)^2 = 2x^3 -4x^2 +8x +7\\Ax^3 +(3A +B +C)x^2 +(4A +4B -2C)x -(8A -8B -C) = 2x^3 -4x^2 +8x +7
So I should be able to conclude that A = 2, however, the problem is that on one hand I get that 3B = -10 and on the other hand, 10B = 23. Have I made a mistake in the calculations? Is any such rational function divisible [not sure if that's the correct word] into partial fractions?
Is there any other method for tackling such a problem?

Thank you in advance.
The last partial fraction should contain an additional first-order term in the numerator: \frac{A}{x-1} +\frac{B}{(x-1)^2} +\frac{Cx+D}{x^2 +4x +8} = \frac{2x^3 -4x^2 +8x +7}{(x-1)^2 (x^2 +4x +8)}
 
Thank you, ehild, for the correction. Could you please explain why the last partial fraction should also contain the first-order term? I do not doubt your words, I can't fully understand the concept myself.
Everything works out nicely, though.
A = 0, B = 1, C = 2 and D = -1 and the rest is trivial.
 
The denominator is of order 2, and you always need one order less in the numerator (for your (x-1)-type fractions, the A serves that purpose).

As a simple example, you cannot express ##\frac{5x+3}{x^2+4x+8}## with ##\frac{C}{x^2+4x+8}##.
 
  • Like
Likes nuuskur
nuuskur said:
Could you please explain why the last partial fraction should also contain the first-order term? I do not doubt your words, I can't fully understand the concept myself.
As you have a third-order polynomial in the numerator, it involves 4 equations when comparing its four coefficients with the expression obtained from the partial fractions. So you need 4 unknowns in general. Otherwise you might arrive at contradiction, as you experienced.
 
  • Like
Likes nuuskur
Question: A clock's minute hand has length 4 and its hour hand has length 3. What is the distance between the tips at the moment when it is increasing most rapidly?(Putnam Exam Question) Answer: Making assumption that both the hands moves at constant angular velocities, the answer is ## \sqrt{7} .## But don't you think this assumption is somewhat doubtful and wrong?

Similar threads

  • · Replies 4 ·
Replies
4
Views
2K
  • · Replies 3 ·
Replies
3
Views
1K
  • · Replies 10 ·
Replies
10
Views
2K
  • · Replies 9 ·
Replies
9
Views
2K
  • · Replies 5 ·
Replies
5
Views
1K
  • · Replies 9 ·
Replies
9
Views
4K
  • · Replies 4 ·
Replies
4
Views
1K
Replies
2
Views
1K
  • · Replies 6 ·
Replies
6
Views
2K
  • · Replies 8 ·
Replies
8
Views
2K